You are on page 1of 75

Introduction Level I & ll Multiple Choice 1. A hemoglobin concentration of 19 g/dL is normal for which individual listed below?

[Hint] 1-day-old infant 5-year-old child 20-year-old male 65-year-old female

2.

If a patient were suffering from appendicitis, you would expect which cells to be increased in the peripheral blood? [Hint] erythrocytes lymphocytes neutrophils platelets

3.

Which cells make up the biggest mass of cells in the peripheral blood? [Hint] erythrocytes platelets neutrophils lymphocytes

4.

These cells are important in maintaining normal hemostasis: [Hint] erythrocytes platelets neutrophils monocytes A patient has a WBC count of 25 x 109/L. The doctor subsequently orders a differential count. This is an example of __________. [Hint] extensive screening doctors error in ordering reflex testing prospective ordering

5.

6.

In a fee-for-service system, laboratory testing is considered ________ to the hospital. [Hint] revenue unnecessary wasteful a cost

7.

You are asked to be a member of a team designing a cost-efficient quality care protocol for patients receiving anticoagulant therapy. This is an example of: [Hint] clinical pathway development

reflexive testing development prospective care protocol disease management

8.

Which of the following is NOT a method to help control costs of medical care? [Hint] clinical pathway development disease management fee-for-service plans reflexive testing

9.

An increase in the concentration of neutrophils in the peripheral blood is most commonly due to: [Hint] blood loss infection poor diet increasing age

10 .

Your laboratory hematology team decides to create a testing protocol that will help physicians identify the cause of anemia. This is an example of: [Hint] reflexive testing capitated testing prospective testing critical pathway development

Cellular Homeostasis and Hematopoiesis Level I Multiple Choice 1. The protein kinase responsible for triggering the sequential steps in the cell-division cycle is: [Hint] cyclin Cdk Cdk inhibitor p21

2.

Apoptosis is best described as: [Hint] pathologic cell death resulting accelerated differentiation programmed cell death terminal differentiation and loss of ability to undergo proliferation

3.

Which of the following are considered to be initiators of apoptosis? [Hint] growth factor withdrawal

loss of attachment to extracellular matrix cell-damaging stress all of the above

4.

All of the following are events regulated by apoptosis EXCEPT: [Hint] elimination of autoreactive lymphocytes elimination of expanded lymphocytes following cessation of immune response elimination of expanded erythrocytes following cessation of hypoxic episode elimination of expanded phagocytic cells following cessation of infection/inflammatory response

5.

Differentiation is best described as: [Hint] a. the appearance of different properties in cells that were initially equivalent the instance when two cells, derived from the same precursor, take separate routes of development the totality of phenomena that begins with commitment and ends when the cell has all of its characteristics The "aging" or senescence of a cell

6.

The morphologically recognizable population of hematopoietic precursor cells, capable of amplification by proliferation, is the: [Hint] mature cells stem cells maturing cells progenitor cells

7.

Hematopoietic growth factors exert all of the following effects on hematopoietic cells EXCEPT: [Hint] promote cell survival by suppressing apoptosis promote cell proliferation regulate the processes of differentiation promote senescence

8.

All of the following are characteristics of hematopoietic growth factors EXCEPT: [Hint] Most growth factors are produced by stromal cells in the microenvironment. Individual growth factors, by themselves, are poor stimulators of colony growth. Most growth factors are relatively lineage-specific. Growth factors commonly act synergistically with other cytokines.

9.

All of the following are considered early-acting (multilineage) growth factors EXCEPT: [Hint] stem cell factor thrombopoietin Flt3 ligand GM-CSF

10 .

A gene with the capacity to transform a cell into a malignant phenotype is a/an: [Hint] proto-oncogene tumor suppressor gene oncogene anti-oncogene

Cellular Homeostasis and Hematopoiesis Level II Multiple Choice 1. The cyclin that is synthesized in response to growth factor stimulation, and is responsible for G 1 progression, is: [Hint] Cyclin A Cyclin B Cyclin D Cyclin E

2.

The cell cycle regulatory protein that functions to inhibit cell cycling by sequestering transcription factors required for cell cycle progression from G1/S is: [Hint] Rb p53 E2F p21

3.

The protein responsible for monitoring for DNA damage, and the integrity of the genome, is: [Hint] Rb p53 E2F p16

4.

The Bcl-2 family of proteins, important in regulating activation of execution caspases, is located: [Hint] free in the cytosol associated with the cellular membrane associated with the mitochondrial membrane associated with the endoplasmic reticulum membrane

5.

Which of the following is considered an example of a death cytokine/death receptor pair? [Hint] kit ligand/kit receptor flt3 ligand/flt receptor fas ligand/fas receptor mpl/mpl receptor

6.

All of the following are considered diseases associated with increased apoptosis EXCEPT: [Hint]

AIDS Parkinsons Disease Systemic lupus erythematosus Aplastic anemia

7.

All of the following are important regulators of thrombopoiesis except: [Hint] TPO IL-11 IL-4 IL-6

8.

The adhesive glycoprotein largely responsible for the retention of developing erythrocyte precursors in the bone marrow is: [Hint] fibronectin hemonectin thrombospondin glycosaminoglycans

9.

A major factor in maintaining stem cell quiescence in the hematopoietic microenvironment is: [Hint] MIP-1 TNF SCF Flt3 ligand

10 .

Proto-oncogenes encode which of the following regulatory proteins? [Hint] growth factors growth factor receptors transcription factors all of the above

Structure and Function of Hematopoietic Organs Level I Multiple Choice 1. The correct sequence of location of hematopoiesis during development is: [Hint] AGM bone marrow liver bone marrow liver spleen yolk sac liver bone marrow yolk sac thymus AGM

2.

All of the following cell types are found in bone marrow except: [Hint] adipocytes

macrophages osteoblasts thymocytes

3.

Which of the following is true of the thymus? [Hint] It enlarges after adolescence. It provides mature macrophages to the rest of the body. It has a high rate of cell death. It is nonfunctional in old age.

4.

Which region of the spleen would most likely be normal in an immune-deficient person? [Hint] white pulp marginal zone germinal center red pulp

5.

Both lymph node and spleen have which structure? [Hint] B-cell follicle marginal zone medulla red pulp

Structure and Function of Hematopoietic Organs Level II Multiple Choice 1. Bone marrow hyperplasia occurs most commonly in association with: [Hint] immune response marrow fibrosis ineffective erythropoiesis toxin exposure

2.

All of the following may cause hypersplenism except: [Hint] metastatic tumor antibody-coated red blood cells clot in the splenic artery cirrhosis

3.

A common cause of lymphadenopathy is: [Hint] infection liver disease

extramedullary hematopoiesis antibody-coated platelets

4.

Which of the following may occur after splenectomy for blood disorders associated with antibodies directed against blood cells? [Hint] decreased platelet count removal of antibodies from circulation increased numbers of spherocytes increased risk of infection

5.

Hereditary spherocytosis may lead to splenomegaly because of: [Hint] increased removal of erythrocytes increased lymphoid tissue the presence of tumor cells the accumulation of undigestible substances

1.

Which of the following stages in erythrocyte development can be found in both the bone marrow and peripheral blood? [Hint] reticulocyte pronormoblast polychromatophilic normoblast orthochromatic normoblast

2.

A NRBC has the following characteristics: no visible nucleoli, moderate N:C ratio, deeply basophilic cytoplasm, irregular clumping of chromatin on the nuclear rim. These characteristics describe which of the following maturation stages? [Hint] orthochromic normoblast basophilic normoblast pronormoblast metarubricyte

3.

The majority (90%) of the erythrocyte's metabolism for energy production is produced from: [Hint] NADH 2,3-DPG glucose glycogen

4.

Predict which of the following erythroid maturation stages would be the most sensitive to erythropoietin stimulation: [Hint] polychromatic normoblast BFU-E reticulocyte CFU-E

5.

The presence of hemosideruria infers which of the following? [Hint]

increased serum iron increased intravascular hemolysis increased serum haptoglobin levels increased serum ferritin

6.

The erythrocyte membrane is important in all the following EXCEPT: [Hint] maintaining erythrocyte flexibility generation of ATP maintaining erythrocyte shape carrier for cell antigens

7.

Which of the following describes normal erythrocytes? [Hint] contain iron granules cell volume between 70 to 100 fL mean diameter of 7 - 8 Life span of 90 days

8.

In extravascular hemoglobin destruction, heme rings are broken down with the resulting direct production of which of the following? [Hint] transferrin conjugated bilirubin carbon monoxide hemopexin

9.

The body's most common and efficient method of removal of aged or abnormal erythrocyte, and recovery of essential components such as iron and amino acids, is: [Hint] intravascular erythrocyte destruction red cell fragmentation and removal in the biliary tract extravascular erythrocyte destruction breakdown of old erythrocytes in the small intestine

10 .

Which of the following describes the reticulocyte ? [Hint] contains RNA reference range is 5-15% has a pyknotic nucleus is smaller than a mature RBC Hemoglobin Level I Multiple Choice 1. Which of the following is a minor hemoglobin component in an adult? [Hint] hemoglobin A

hemoglobin A2 hemoglobin Portland hemoglobin Gower I

2.

The Bohr effect refers to the ability of hemoglobin to: [Hint] accept H+ carry CO2 exchange Cl- for HCO3respond to changes in 2,3-DPG levels

3.

What effect would iron deficiency have on the rate of hemoglobin synthesis? [Hint] increase it decrease it no effect cant be predicted

4.

When 2,3-DPG increases, the oxygen affinity of hemoglobin: [Hint] increases is unaffected decreases is unpredictable

5.

How many iron molecules are present in a molecule of hemoglobin? [Hint] one two three four

6.

An increase in the concentration of HbA1c is an indication of: [Hint] iron deficiency methemoglobinemia diabetes increased HbA

7.

Which of the following shifts the ODC to the right? [Hint] decreased PCO2

increased PO2 decreased H+ concentration increased temperature

8.

Which of the following hemoglobins is the predominant hemoglobin in infants? [Hint] hemoglobin A hemoglobin A2 hemoglobin F hemoglobin Portland

9.

Normally, hemoglobin concentration is maintained in a steady state by: [Hint] balance in the production and destruction of erythrocytes coordination of production of equal quantities of and globin chains regulation of transferrin receptors regulation of destruction of ferritin mRNA

10 .

A hemoglobin of 160 g/L in an adult male is considered: [Hint] decreased normal increased borderline abnormal

Hemoglobin Level II Multiple Choice 1. Which of the following is the major hemoglobin found in adults? [Hint] 2 2 2 2 2 2 2 2

2.

Insertion of iron into the protoporphyrin ring takes place in the: [Hint] cytoplasm ribosomes mitochondria nucleus

3.

Hemoglobin electrophoresis should be considered when a patient has: [Hint] methemoglobinemia

carboxyhemoglobinemia sulfhemoglobinemia elevated glucose level

4.

A patient has lost blood due to chronic intestinal bleeding, and has no symptoms of anemia although his hemoglobin is 9 g/dL. The best explanation for this is: [Hint] His ODC curve is shifted left. His ODC is shifted right. The lab result is in error. He has increased synthesis of HbF.

5.

A one-month-old baby was cyanotic. The methemoglobin level is 12%. Hemoglobin electrophoresis is normal. What additional test should be done on this baby? [Hint] NADPH reductase activity reticulocyte count isoelectric focusing Hemoglobin A1C determination

6.

A patient with emphysema is treated with long term O2 therapy. Why does this result in an increase in the amount of O2 delivered to the tissues? [Hint] The synthesis of erythrocytes is increased. The synthesis of 2,3-DPG is enhanced. It decreases the amount of O2 expired. It increases the PO2 of inspired air.

7.

What effect does the release of O2 in the capillaries have on the H+ concentration in tissues? [Hint] increases it decreases it has no effect variable effect

8.

You are a healthy male from New York who normally does 60 minutes of aerobic exercise daily. While on vacation, you climb to 8,000 feet above sea level at a mountain resort. Your heart rate is rapid and you are breathing fast. You feel dizzy. What is the best explanation for this set of symptoms? [Hint] the atmospheric PO2 is increased the atmospheric PO2 is decreased your hemoglobin content is decreased you have hypothermia

9.

In iron deficiency, you would expect which of the following to occur: [Hint] increased synthesis of ferritin decreased synthesis of transferrin receptors

decreased ALAS activity increased synthesis of globin chains

10 .

In a child with hemoglobin M, you would expect to find: [Hint] abnormal NADPH reductase activity shift to the right in ODC shift to the left in ODC an improvement after treatment with ascorbic acid

The Leukocyte Level l Multiple Choice 1. Diapedesis refers to cell movement: [Hint] through the blood stream through endothelial cells in the tissues in the bone marrow

2.

The first morphologically identifiable cell in the granulocytic series is: [Hint] CFU-GM cell CFU- G progenitor cell CD34+ cell myeloblast

3.

Traditionally, the hallmark of differentiating myeloblasts from promyelocytes morphologically has been the visual identification of ____ in the promyelocytes: [Hint] primary granules secondary granules loss of nucleoli pink cytoplasm

4.

Peroxidase containing granules are present in the following granulocytes: [Hint] only myeloblasts only promyelocytes only promyelocytes and myelocytes promyelocytes, myelocytes, metamyelocytes, band and segmented neutrophils

5.

The cells that are polymorphonuclear include: [Hint] neutrophils monocytes and neutrophils neutrophils, basophils, and eosinophils

neutrophils, monocytes, and lymphocytes A neutrophils primary function is to: [Hint] protect the host from infectious agents protect the host from autoimmune destruction secrete cytokines to attract monocytes to the site of infection secrete cytokines to attract lymphocytes to the site of infection

6.

7.

The leukocytes responsible for phagocytosis include: [Hint] lymphocytes and monocytes lymphocytes and macrophages monocytes and neutrophils eosinophils and basophils

8.

An adult reference range for monocytes is: [Hint] 0.0 - 0.45 x 109/L 0.2 - 0.8 x 109/L 0.2 - 4.0 x 109/L 0.2 - 7.0 x 109/L Calculate the absolute number of neutrophils given the following information: Total white count = 10 x 10 9/L. Total RBC count = 5.0 x 1012/L, Neutrophils = 40%, metamyelocytes = 30%, myelocytes = 20%, and lymphocytes = 10%. [Hint] 4 x 103/L 2 x 109/L 4 x 109/L 20 x 109/L

9.

10 .

Causes of increased lymphocytes include: [Hint] hypersensitivity reactions eosinophilia and viral infections viral infections tuberculosis, Hodgkins, and nonhematologic malignancies

11 .

Physiologic variations that affect the leukocyte concentration include all of the following except: [Hint] pregnancy time of day race weight and height The newborn peripheral blood white cell differential, contrasted to that of adults, is the following finding in a newborns smear: [Hint] leukopenia

12 .

leukocytosis neutropenia monocytosis

13 .

Immunoglobulin diversity is provided by: [Hint] rearrangement of the immunoglobulin light and heavy chains gene segments in B -lymphocytes selective translation of portions of immunoglobulin mRNA in the B-lymphocyte rearrangement and recombination of gene coding sequences for immunoglobulin in the T-lymphocyte rearrangement and recombination of the VH, DH, and JH genes in the T-lymphocyte

14 .

Causes of monocytosis include: [Hint] hypersensitivity reactions eosinophilia and viral infections viral infections and Bordetella pertussis infections tuberculosis, Hodgkins, and nonhematologic malignancies

15 .

Physiologic variations that affect the leukocyte concentration include all of the following except: [Hint] gender age activity level platelet count The most striking difference, when contrasting infants peripheral blood white cell differential to that of adults, is the following finding in an infants smear: [Hint] neutrophilia eosinophilia monocytosis lymphocytosis

16 .

17 .

Immunoglobulin diversity allows the: [Hint] B-lymphocyte to produce a variety of specific immunoglobulins that can react with foreign antigens. selective translation of portions of immunoglobulin mRNA in the B-lymphocyte rearrangement and recombination of gene coding sequences for immunoglobulin in the T-lymphocyte rearrangement and recombination of the TCR genes in the T-lymphocyte

Routine Hematology Procedures Level I Multiple Choice Top of Form 1. Which anticoagulant prevents coagulation by inhibiting thrombin? [Hint] lithium heparin

sodium citrate EDTA sodium fluoride

2.

When collecting a series of tubes during a phlebotomy, which specimen collection tube should be filled last? [Hint] green top red top blue top lavender top

3.

The 1992 OSHA Bloodborne Pathogens Standard did NOT require laboratory facilities to: [Hint] supply phlebotomists with gloves implement the use of sharps containers mandate vaccination for HCV provide training in potential risk of bloodborne pathogens

4.

Which of the following will NOT affect the phlebotomist's choice of venipuncture site? [Hint] presence of an intravenous line large hematoma invisible but palpatable vein previous mastectomy on left side

5.

What is the function of a microscope's objective? [Hint] magnify the image to the eyepieces collect the diffracted light from the condenser direct the light beam onto the specimen project image to the field lens

6.

An achromat lens will correct: [Hint] chromatic aberrations at two colors and field curvature spherical aberrations at two colors and chromatic aberrations at two colors chromatic aberrations at two colors and spherical aberrations at one colors spherical aberrations at three colors and field curvature

7.

A manual leukocyte count was performed on an EDTA-anticoagulated specimen. The specimen was diluted 1:20 and a total of 132 leukocytes were counted in the four corner squares of the hemacytometer. What is the leukocyte count? [Hint] 2.6 x 109/L 5.3 x 109/L 6.6 x 109/L

11.0 x 109/L The following erythrocyte data were obtained from an EDTA-anticoagulated specimen: erythrocyte count = 3.92 x 1012/L, hemoglobin = 11.4 g/dL, hematocrit = 34% (.34 L/L). Calculate the MCH. [Hint] 29.1 pg 33.5 pg 67.2 pg 86.7 pg

8.

9.

Which diluent is used for manual platelet counts? [Hint] 1% phyloxine B 3% acetic acid 0.85% sodium chloride 1% ammonium oxalate

10 .

A reticulocyte count was performed using a Miller disk, and 30 reticulocytes (square A) were observed in 290 erythrocytes (square B). What is the reticulocyte count? [Hint] 0.9% 1.1% 2.3% 3.0%

11 .

Which of the following will be observed if a purple top collection tube is underfilled? [Hint] falsely elevated leukocyte count falsely decreased erythrocyte sedimentation rate falsely elevated hemoglobin falsely decreased reticulocyte count

12 .

A patient has an elevated fibrinogen level. How will this affect the patient's ESR? [Hint] ESR will be unaffected. ESR will be elevated. ESR will be decreased. ESR will be uninterpretable.

13 .

The clinical laboratory scientist overfilled the second side of the hemacytometer, but carefully removed excess fluid with a kimwipe. What effect would this have on the hemacytometer cell count? [Hint] Result would be unaffected. Result would be falsely decreased. Result would be falsely increased. Result would be equivocal.

14 .

In performing an examination on a Wright-stained peripheral blood smear, the clinical laboratory professional observes enlarged platelets and vacuolated leukocytes. What is the appropriate course of action? [Hint]

Perform a second 100-cell differential. Ignore them, since they are artifacts, and report platelet and leukocyte morphology as normal. Collect new specimen and prepare blood smear within three hours. Send smear for pathology review, since these changes are abnormal. The following erythrocyte data were obtained from an EDTA-anticoagulated specimen: erythrocyte count = 2.93 x 1012/L, hemoglobin = 5.9 g/dL, hematocrit = 21% (.21 L/L). What would you expect to observe on a Wright-stained peripheral blood smear? [Hint] normochromic, normocytic erythrocytes hypochromic, microcytic erythrocytes normochromic, macrocytic erythrocytes hypochromic, normocytic erythrocytes

15 .

16 .

A patient is known to have high levels of sulfhemoglobin. How will this affect the patient's hemoglobin determination? [Hint] Result will be unaffected, since sulfhemoglobin is measured by the cyanmethemoglobin method. Result will be falsely elevated, since sulfhemoglobin will be precipitated, causing turbidity. Result will be falsely decreased, since sulfhemoglobin is not measured by the cyanmethemoglobin method.. Result must be corrected for the presence of sulfhemoglobin using a correction factor.

Bottom of Form Peripheral Blood Smear Level I Multiple Choice Top of Form 1. The microscopic examination of a Wright-stained blood smear revealed bluish-gray erythrocytes and intensely black leukocyte nuclei. What is the best explanation for this appearance? [Hint] The buffer was too acidic. The rinsing process was prolonged. The blood smear was too thin. The Wright stain was too alkaline.

2.

In the examination of a blood smear, 95 leukocytes were observed in five fields of view. What is the leukocyte estimate? Assume an EDTA blood specimen was used to prepare the smear. [Hint] 1.9 x 109/L 3.8 x 109/L 19.0 x 109/L 38 x 109/L If the leukocyte count in question #2 was 4.5 x 109/L, does the leukocyte estimate correlate? [Hint] yes no A leukocyte count of 28.5 x 109/L was obtained on an EDTA-anticoagulated specimen. Examination of the blood smear revealed 112 nucleated erythrocytes per 100 leukocytes. What is the corrected leukocyte count? [Hint] 13.4 x 109/L

3.

4.

16.8 x 109/L 25.4 x 109/L 28.5 x 109/L Which component of Wrights stain is responsible for the red-orange color of an erythrocyte? [Hint] azure B methylene blue eosin methanol

5.

6.

A patient's CBC results included an MCV = 72 fL, MCH = 30 pg, and MCHC = 33%. What would you expect to observe for the erythrocyte morphology? [Hint] normochromic and normocytic normochromic and microcytic normochromic and macrocytic hypochromic and microcytic

7.

If you observe platelet satellitism on the Wright-stained blood smear of a patient, what CBC parameter should you check for accuracy? [Hint] platelet count WBC count RBC count reticulocyte count

8.

You count 15 platelets in each of five microscopic fields at 100x magnification on a Wright stained blood smear. What is the approximate platelet count? [Hint] 500 x 109/L 225 x 109/L 100 x 109/L 23 x 109/L

9.

You count about six platelets per field at 100x magnification on a Wright-stained smear. What is the most accurate description of the platelet count? [Hint] normal decreased increased cant tell from this information

10 .

One day the clinical laboratory technician notices that all blood smears examined under the microscope show crenated red blood cells. What is the potential problem ? [Hint] The smears are too thin. The room is too cold.

The smear is too thick. The smears are dried too slowly. Bottom of Form Peripheral Blood Smear Level II Multiple Choice 1. A patient with a history of polycythemia is seen for a routine checkup. What might the clinical laboratory personnel have to do to prepare an optimal blood smear? [Hint] Decrease the angle of the spreader slide and decrease the push speed. Decrease the angle of the spreader slide and increase the push speed. Increase the angle of the spreader slide and increase the push speed. Increase the angle of the spreader slide and decrease the push speed.

2.

Initial examination of a Wright-stained blood smear reveals bright red erythrocytes and very pale leukocyte nuclei. To correct this improper staining, the clinical laboratory professional should: [Hint] Decrease the staining time. Prepare a thinner blood smear. Decrease the pH of the stain. Increase the pH of the buffer

3.

In examining the patient's Wright-stained blood smear, the clinical laboratory professional noted the presence of platelet clumps. How would an accurate platelet estimate be obtained on this patient? [Hint] Warm specimen to 37oC for 15 minutes, then prepare peripheral blood smear. Allow specimen to set at 25oC for five hours, then prepare peripheral blood smear. Recollect specimen in citrate and prepare peripheral blood smear. No corrective action is available.

4.

A peripheral blood smear examination of a Wright-stained blood smear revealed leukocytes with cytoplasmic vacuolization and karyorrhexis of the nucleus. What action should be taken to determine if these changes are true or artifact? [Hint] Prepare and stain a second smear after blood has stabilized for four hours. Recollect the specimen and prepare blood smear immediately. Mix a portion of the blood with 22% albumin and prepare a blood smear. Collect specimen using heparin and prepare a blood smear. A patient has cold agglutinin disease. How can a good blood smear be made from this patients blood? [Hint] Dry the smear quickly. Decrease the angle of the spreader slide. Press hard on the spreader slide Warm the blood to 37 degrees C before making the smear.

5.

6.

A new employee complains that the staining procedure is washing all the blood off the slide. What is the likely problem? [Hint] The stain is old. The blood smear is too thick.

The step with methanol is being skipped. The buffer is too acid. You want to determine if a patients platelet count is falsely decreased due to platelet clumping. What is the most appropria te action to take? [Hint] Examine the stained blood smear at 1000X. Examine the stained blood smear at 100X. Shake the EDTA tube vigorously and reanalyze. Collect the blood in lithium heparin and reanalyze.

7.

8.

You note rouleaux on the stained periperal blood smear. How will this affect the CBC? [Hint] This will not affect the CBC. The WBC count will be falsely decreased. The RBC count will be falsely decreased. The platelet count will be falsely increased.

9.

You are doing a differential count on a peripheral blood smear and note that there are many smudge cells present. What should you do? [Hint] Add 1 drop of saline to a drop of blood and make another smear. Add a lysing agent to the patients blood and make another smear. Add 1 drop of albumin to 5 drops of blood and make another smear. Recollect the blood in citrate and make another smear.

10 .

A renal dialysis patient has a hematocrit of 15%. What adaptation can you make to assure a good blood smear? [Hint] Increase the angle of the spreader slide and push speed. Decrease the angle of the spreader slide and push speed. Increase the angle of the spreader slide and decrease push speed. Decrease the angle of the spreader slide and decrease push speed.

Bone Marrow Level I Multiple Choice 1. What is the preferred site for bone marrow aspiration in adults? [Hint] posterior iliac crest vertebral bodies tibia sternum

2.

Which of the following are indications for a bone marrow biopsy? [Hint] 35-year-old HIV-positive male with fever of unknown origin 60-year-old male with anemia secondary to malignancy 20-year-old woman with iron deficiency

a child with infectious mononucleosis

3.

The anteromedial surface of the tibia is sometimes used for bone marrow aspiration in: [Hint] patients more than 60 years old children less than 2 years old women in their 30s adults

4.

Which of the following is not an indication for a bone marrow biopsy? [Hint] Pancytopenia with rare circulating blasts Thrombocytopenia with rare circulating blasts Mild leukocytosis after receiving growth factor Unexplained anemia and leukopenia

5.

Which bone marrow sample is best for evaluating cell morphology and performing the differential count? [Hint] aspirate biopsy clot section touch prep

6.

The normal M:E ratio ranges between: [Hint] 1: 2 to 1: 4 2: 1 to 4: 1 1: 4 to 1: 6 4: 1 to 6: 1

7.

A pathologist examined the core biopsy of a 50-year old male. The overall cellularity is 20%. Interpret this finding. [Hint] Normocellular marrow for age Hypercellular marrow for age Hypocellular marrow for age You cannot estimate from a core biopsy.

8.

A physician is evaluating a patient with systemic lupus erythematosus. The patient is anemic. There is difficulty interpreting the serum iron studies. The anemia could be due to chronic disease secondary to lupus or iron deficiency. A bone marrow biopsy was performed. What special stain should be used to evaluate the marrow? [Hint] myeloperoxidase Sudan black B Prussian blue new methylene blue

9.

A physician did a bone marrow biopsy in a patient with previous diagnosis of primary myelofibrosis. He wants to evaluate the degree of fibrosis. What stain should be used?[Hint]

Reticulin Prussian blue myeloperoxidase TdT

10 .

Why would special stains such as myeloperoxidase be performed on a bone marrow specimen? [Hint] to differentiate types of anemia to differentiate types of leukemia to evaluate cellularity to determine M:E ratio

Bone Marrow Level II Multiple Choice 1. A 20-year-old male with suspicion of acute leukemia had a bone marrow biopsy. His platelet count is 30 x 10 9L. The man is oozing from the biopsy site. What should be done? [Hint] Request platelets from the blood bank for platelet transfusion. Transfuse the patient with fresh frozen plasma. Apply local pressure and ask the patient to lie on his back. Watch the bleeding site periodically. Give factor VIII concentrate.

2.

You are in the laboratory to process a bone marrow biopsy. The patient is suspected to have acute leukemia. The physician who performed the bone marrow couldnt get any aspirate. You see several unstained slides that appear to be bone marrow imprints. What is the b est way to use these slides? [Hint] Stain all slides with Wright-Giemsa. Stain a few slides with Wright-Giemsa; save the unstained slides for possible cytochemical stains. Stain a couple of slides with Wright-Giemsa. You cannot perform cytochemical stains on bone marrow imprints. Perform routine cytogenetics on unstained slides.

3.

A patient is suspected of having an acute leukemia. His bone marrow revealed 40% blasts. What ancillary study can be done on the bone marrow aspirate in almost any laboratory to determine the lineage of the blasts? [Hint] molecular diagnostics cytogenetics flow cytometry cytochemical stains

4.

A pathologist is evaluating a bone marrow aspirate performed on a 10-month-old boy who required a bone marrow for a non-hematolymphoid malignancy. He sees an increased number of small, lymphoid-appearing cells with high nuclear cytoplasmic ratio and fine chromatin that look similar to blasts. What are these cells? [Hint] mature lymphocytes hematogones: normal lymphoid progenitor cells immature megakaryocytes normal erythroid precursors

5.

The following is the bone marrow differential obtained in a patient with a recent diagnosis of chronic myeloid leukemia: Blasts 2%, promyelocytes 10%, myelocytes 40%, metamyelocytes 20%, bands 5%, segmented neutrophils 10%, basophils 3%, pronormoblasts 2%, basophilic normoblasts 3%, polychromatophilic normoblasts 3%, orthochromatophilic normoblasts 2%. What is the M:E ratio? [Hint] 9:1 11:1 10:1 5:1

6.

What is the overall bone marrow cellularity expected in a 30-year-old woman? [Hint] 100% 60 to 80% 20% 10%

7.

A pathologist is evaluating a bone marrow and saw morphologic features suggestive of a myelodysplastic syndrome. She wants to know if these are ringed sideroblasts. What sample is not suitable to evaluate ringed sideroblasts?" [Hint] aspirate smear core biopsy clot preparation particle smears

8.

A 50-year-old male with neutropenia and splenomegaly had a bone marrow biopsy. The bone marrow revealed 60% blasts. On the peripheral blood smear are several suspicious, blast-like cells. The diagnosis is leukemia. Why do ancillary studies need to be done? [Hint] to determine the lineage of the blasts to determine if the leukemia is secondary to exposure to toxins to determine the presence or absence of oncogenes to get a more accurate blast count in the bone marrow

9.

You are assisting a physician with a bone marrow biopsy. The patient is a 30-year-old male suspected of having acute leukemia. He was able to obtain a good bone marrow aspirate and a good core biopsy. He wants to make sure you send material for flow cytometry and cytogenetics. What is the best way to divide the material you have for the different labs? [Hint] Make several smears out of the aspirate for morphology and possible special stains. Allocate the remaining aspirate material into two tubes, an EDTA tube for flow cytometry and a heparin tube for cytogenetics. Fix the entire core biopsy in formalin. Use all aspirate, making smears for morphology and special stains. Divide the core biopsy in two and send one piece for the flow lab and one piece for the cytogenetics lab. Allocate the aspirate material in two EDTA tubes, one for the flow lab and one for cytogenetics. Dont make aspirate smears. P lace the core biopsy in formalin. None of the above is correct.

10 .

A patient has had several bone marrow biopsies. The number of lymphocytes varied significantly from one specimen to another. What could be the possible explanation for this? [Hint] The patient had a viral infection during one of the procedures. A lymphoid follicle may have been aspirated in one procedure. There is a mix-up of patient specimens. Not enough cells were counted in the differential to get a good distribution.

1.

A cell with an MCV of 69 fL is called a(n): [Hint]

anisocyte normocyte macrocyte microcyte

2.

Classify the cells with the following indices: MCV 106 fL, MCH 35, MCHC 33. [Hint] macrocytic, hypochromic macrocytic, normochromic normocytic, normochromic microcytic, hypochromic Which of the indices best fit the following results? RBC count 3.0 x 1012/L; hemoglobin 90 g/L; hematocrit 0.25L/L. [Hint] macrocytic, normocytic microcytic, hypochromic normocytic, normochromic microcytic, normochromic

3.

4.

What is the RPI given the following? Hematocrit 0.33 L/L, reticulocyte count 10%. [Hint] 4.8 3.7 2.9 9.0

5.

Which of the following is a characteristic finding on the blood smear of a specimen that has a reticulocyte count of 21.5%? [Hint] hypochromic macrocytes polychromatophilic macrocytes Howell-Jolly bodies basophilic stippling

6.

A patient has anemia with the following indices: MCV 64 fL., MCH 20.5 pg, MCHC 31.8 g/dL. What is the morphologic classification of this anemia? [Hint] macrocytic, hyperchromic normochromic, normocytic normocytic hypochromic microcytic, hypochromic

7.

Upon review of the blood smear of a patient with sideroblastic anemia, erythrocytes that contain small irregular basophilic deposits are noted. Another slide is made and stained with Prussian blue. The deposits are also seen with this stain. What are these intracellular deposits? [Hint] Howell-Jolly bodies Heinz bodies

Pappenheimer bodies basophilic stippling

8.

Which of the following tests is helpful in evaluating the rate of erythrocyte destruction? [Hint] bilirubin MCV hemoglobin serum iron What is the absolute reticulocyte count if the RBC count is 4 x 1012/L and the reticulocyte count is 15%? [Hint] 600 x 109/L 60 x 109/L 27 x 109/L 38 x 109/L

9.

10 .

Upon review of a stained blood smear you note that erythrocytes appear to form irregular clusters, like grapes. How would you report this? [Hint] RBC agglutination noted rouleaux present grape cells present cold agglutinins present

Introduction to Anemia Level II Multiple Choice 1. A patient with anemia has an RDW of 14.0 and an MCV of 120 fL. However, examination of the blood smear reveals significant anisocytosis. How do you explain these results? [Hint] The high MCV causes the falsely normal RDW. The instrument needs to be recalibrated. The MCV is falsely elevated. The RDW is only elevated if there is anisocytosis and a normal MCV.

2.

Upon review of a stained blood smear, you note that erythrocytes appear to form irregular clusters, like grapes. What CBC parameter is unreliable based on this observation? [Hint] WBC count platelet count hemoglobin MCV

3.

A male patient has a hemoglobin of 50 L/L and a hematocrit of 0.14 L/L. His reticulocyte count is 25%. There are numerous polychromatophilic erythrocytes on the blood smear. How would you describe this patients bone marrow response? [Hint]

hypoproliferative bone marrow response absent bone marrow response compensatory bone marrow response impaired erythropoietic response

4.

Upon review of a stained blood smear, you note that most erythrocytes appear to be stacked on one another. What laboratory test should you look up on this patient to help verify that this is not an artifact? [Hint] total protein haptoglobin vitamin B12 serum iron A 6-year-old patient has a hemoglobin of 8.0 g/dL, RBC count 3.83 x 1012/L, hematocrit 0.25 L/L, RDW 15, platelet 156 x 109/L, WBC 10 x 109/L. What is the morphologic classification of this anemia? [Hint] macrocytic microcytic, hypochromic normocytic, normochromic survival defect

5.

6.

What laboratory test should be done on the above patient to help determine the functional classification of the anemia? [Hint] haptoglobin reticulocyte count bone marrow examination serum ferritin

7.

A patient has the following results: Hemoglobin 90 g/L, hematocrit 0.29 L/L, RBC count 2.6 x 1012/L, reticulocyte count 12%. What is the morphologic and functional classification of this anemia? [Hint] normocytic, survival defect macrocytic, non-megaloblastic defect macrocytic, survival defect macrocytic, nuclear maturation defect

8.

What laboratory test is essential to determine if an anemia is due to hemolysis? [Hint] serum ferritin reticulocyte count vitamin B12 RDW Upon review of a Wrights-stained blood smear, you note small irregular

9.

basophilic inclusions in the erythrocytes. You are unsure as to whether this is basophilic stippling or Pappenheimer bodies. How would you differentiate these two? [Hint] Stain another blood smear with Prussian blue. Review the Wrights-stained smear with phase microscopy. Examine the Wrights-stained smear with polarized light. Stain the blood with new methylene blue

10 .

A patient has a macrocytic anemia. A bone marrow aspirate is obtained and examined. The marrow is 60% cellular and the M:E ratio is 1: 2. These findings indicate: [Hint] hypoplasia increased fat to cell ratio increased erythropoiesis dysplasia

nemias of Disordered Iron Metabolism and Heme Synthesis Level I Multiple Choice 1. What is the most common cause of iron deficiency in adult males in developed countries? [Hint] poor diet chronic blood loss malabsorption increased iron requirement

2.

This hereditary disease is characterized by an increase in total body iron and no anemia or abnormal RBC morphology: [Hint] hemochromatosis IDA ACD porphyria

3.

Which form of iron can be visualized by the Prussian blue stain? [Hint] transferrin ferritin heme hemosiderin

4.

Which of the following anemias is caused by a gene mutation? [Hint] iron-deficiency sideroblastic hemochromatosis anemia of chronic disease

5.

A patient has a normal MCV, MCH, and MCHC, but examination of the peripheral blood smear reveals hypochromic and normochromic cells. Which set of iron study results would you expect on this patient? [Hint] serum iron decreased, transferrin saturation increased serum iron increased, transferrin saturation decreased serum iron increased, transferrin saturation increased serum iron normal, transferrin saturation normal

6.

Microcytic, hypochromic erythrocytes are most characteristic of which of the following anemias? [Hint] anemia of chronic disease sideroblastic anemia porphyria iron-deficiency anemia

7.

Cytokines are implicated in the pathophysiology of which anemia? [Hint] anemia of chronic disease sideroblastic anemia porphyria iron-deficiency anemia

8.

Which of the following is most closely associated with TIBC? [Hint] transferrin hemosiderin ferritin hemoglobin A patient has a TIBC of 330 g/dL and a serum iron of 50 g/dL. What is the transferrin saturation? [Hint] 30% 50% 66% 15%

9.

10 .

Which of the following parameters may be abnormal in a non-anemic individual who is iron-deficient? [Hint] hemoglobin MCHC reticulocyte hemoglobin content (CHr) RBC count

Hemoglobinopathies: Qualitative Defects Level I Multiple Choice 1. The basic defect resulting in production of qualitatively abnormal hemoglobins involves: [Hint]

reduced synthesis of beta globin chains reduced synthesis of heme reduced synthesis of alpha globin chains amino acid substitutions or deletions in globin chains

2.

The defect resulting in production of HbS is: [Hint] 6 (A3) Glu > Val 6 (A3) Glu > Lys 121 (GH4) Glu > Gly 26 (B8) Glu > Lys

3.

Laboratory findings for persons with homozygous HbE include: [Hint] Markedly decreased hemoglobin, crystals within erythrocytes mildly decreased hemoglobin, decreased MCV, target cells approximately equal amounts of HbE and HbA on electrophoresis crystals within erythrocytes and normal MCV

4.

Laboratory findings for persons with HbC disease include: [Hint] markedly decreased hemoglobin, crystals within erythrocytes elevated hemoglobin and hematocrit, decreased MCV, target cells crystals within erythrocytes and erythrocytes that are folded or contracted approximately equal amounts of HbC and HbA on electrophoresis

5.

Laboratory findings for persons with sickle cell disease include: [Hint] markedly decreased hemoglobin, target cells on smear, normal MCV and MCHC mildly decreased hemoglobin, decreased MCV, normal reticulocyte count approximately equal amounts of HbS and HbA on electrophoresis normal hemoglobin, boat-shaped cells on smear, increased MCHC

6.

Which hemoglobinopathy results in the most severe clinical disorder? [Hint] HbS/HbS HbS/HbA HbC/HbC HbE/HbE

7.

Laboratory findings for persons with HbC trait include: [Hint] markedly decreased hemoglobin, crystals within erythrocytes

elevated hemoglobin and hematocrit, increased MCV crystals within erythrocytes and erythrocytes that are folded or contracted approximately 30% to 40% HbC on electrophoresis, target cells on smear

8.

The solubility test is a rapid test that will detect the presence of abnormal hemoglobin in which condition? [Hint] HbS/HbS only HbS/HbS and HbA/HbS HbE/HbC HbC/HbC

9.

The sodium metabisulfite test is useful in: [Hint] rendering HbS insoluble, resulting in an opaque appearance causing cells containing HbS to sickle separating hemoglobins based on electrical charge quantitating HbF

10 .

The presence of many abnormal hemoglobins can be confirmed by this laboratory test, which depends on differences in the charge of the hemoglobin molecule: [Hint] hemoglobin solubility test hemoglobin heat instability hemoglobin electrophoresis determination of P50 of hemoglobin

Thalassemia Level I Multiple Choice 1. Alpha thalassemia is best defined as: [Hint] a reduction in the synthesis of -globin chains due primarily to -gene deletions an amino acid substitution in the globin chain that affects hemoglobin solubility characteristics an increase in the synthesis of -chains that precipitate in the erythrocytes and produce chronic hemolytic anemia a loss of suppression of -chain synthesis resulting in excessive production Hgb F How many -globin alleles are affected in -thalassemia minor? [Hint] 1 2 3 4

2.

3.

Select the hemoglobin that is typically the major hemoglobin found in infants with hydrops fetalis: [Hint]

Hb A Hb Lepore Hb Barts Hb F

4.

Alpha thalassemia silent carrier could best be described by which of the following genetic designations? [Hint] homozygous -thal 1 heterozygous -thal 1/normal heterozygous -thal 1/+/thal 2 heterozygous +-thal 1/normal Select the laboratory test result that is consistent with a diagnosis of -thalassemia minor: [Hint] positive hemoglobin solubility test microcytic anemia absence of Hb A sickle cells present Which of the following -thalassemias is typically asymptomatic? [Hint] -thalassemia minor Cooleys anemia Hb H disease Hydrops fetalis

5.

6.

7.

Of the following genotypes, which is associated with -thalassemia major? [Hint] +/ / / /

8.

An adult immigrant patient suspected of having thalassemia presents with the following hemoglobin electrophoresis pattern: Hb A = 0%, Hb F = 90%, Hb A2 = 10%. This pattern best fits a diagnosis of: [Hint] Hb H disease -thalassemia major -thalassemia trait -thalassemia minor

9.

In which ancestry is the more severe version of -thalassemia major most often encountered? [Hint] African Central European

North American Mediterranean

10 .

The chronic hemolysis in -thalassemia major contributes to all of the following EXCEPT: [Hint] gallstones anemia target cells splenomegaly

Thalassemia Level II Multiple Choice 1. High levels of Hgb F can be found in the following: [Hint] HPFH hydrops fetalis 1-year-old normal child Hb H disease

2.

Which of the following is a genetic mutation found in thalassemia? [Hint] nonsense mutation of the globin gene splice site mutation of ALAS2 gene amino acid substitution in globin chain deletion of transferrin gene

3.

Which two of the following thalassemias produce the greatest degree of Heinz body formation? [Hint] Hb H disease and -thalassemia major hydrops fetalis and -thalassemia minor -thalassemia minor and -thalassemia minor Hb H disease and silent carrier Excess -chains in -thalassemia major are responsible for which of the following pathologies: [Hint] presence of Hb Barts elevated Hb H levels Heinz body formation decreased HbA

4.

5.

Which of the following is a characteristic of Hb H? [Hint] It is unstable and tends to precipitate inside erythrocytes. It has a lower affinity for oxygen than Hb A. It has a higher P50 value on hemoglobin dissociation curves than Hb A.

It is composed of a tetramer of -chains. The /-chain ratio in a patient with -thalassemia major is expected to be: [Hint] 1:1 <0.25 1:2 >1.0 Which statement best explains the mechanism of two patients with the genotype + + being diagnosed with two different clinical forms of thalassemia, -thalassemia major and -thalassemia intermedia? [Hint] Their immune systems are genetically different enough to alter the impact of the pathology on the symptoms. One patient presents with both -gene deletions on the same chromosome and the other with one -gene deletion on each of two chromosomes. The + + mutation in -thalassemia may result in different clinical diseases depending on degree of -chain production. The increased expression of Hb F in -thalassemia is variable and results in different clinical diseases.

6.

7.

8.

HPFH is generally asymptomatic because: [Hint] Hb F possesses oxygen-binding and -release characteristics similar to Hb A. Hb A levels are near normal even in the homozygotes. Hb H and Hb Barts compensate for the reduction in Hb F levels. The disease can be adequately treated with desferrioxamine.

9.

Which of the following thalassemias represents deletion or inactivation of the entire -gene complex? [Hint] -thalassemia major /-thalassemia -thalassemia HPFH

10 .

Which laboratory test is most useful at differentiating the variants of HPFH? [Hint] Kleihauer/Betke stain Brilliant Cresyl Blue hemoglobin solubility hemoglobin electrophoresis on cellulose acetate

11 .

A 37-year-old patient with microcytic, hypochromic anemia has the following hemoglobin electrophoresis results: Hb A = 97%, Hb F = 1%, Hb A2 = 2%. Assuming only the CBC and hemoglobin electrophoresis were done, what additional laboratory studies should done on this patient? [Hint] serum iron, serum ferritin Heinz body preparation lead level measurement bone marrow examination

Megaloblastic and Nonmegaloblastic Macrocytic Anemias

Level I Multiple Choice 1. A deficiency in vitamin B12 or folic acid results in ineffective erythropoiesis because: [Hint] Both vitamin B12 and folic acid are necessary cofactors in the anabolic pathway that synthesizes DNA thymidine. B12 is necessary to bind folic acid for transport to the erythroid marrow. Both B12 and folic acid are necessary for the uptake of iron into erythroid cells. Both B12 and folic acid are necessary for the uptake of iron into erythroid cells. Folic acid and B 12 are vital for the proper synthesis of erythropoietin by the kidneys. 2. Pernicious anemia is a megaloblastic anemia resulting from: [Hint] dietary deficiency of vitamin B12 folate deficiency a deficiency in the amount or function of intrinsic factor hepatic cirrhosis

3.

A urinary methylmalonic acid determination is a sensitive diagnostic test for: [Hint] folic acid deficiency intrinsic factor deficiency vitamin B12 deficiency homocysteine increase

4.

Which of the following conditions is an acquired deficiency of intrinsic factor function? [Hint] autoantibodies to parietal cells gastrectomy autoantibodies to the IF-B12 complex bowel resection

5.

A hemolytic anemia can become macrocytic if accompanied by: [Hint] reticulocytosis target cells spherocytes elliptocytes

6.

All of the following are typical RBC morphological findings for a person with megaloblastic anemia EXCEPT: [Hint] Burr cells Howell-Jolly bodies NRBCs macroovalocytes

7.

Macrocytosis with a megaloblastic marrow can be accompanied by which of the following diagnostic characteristics? [Hint] pancytopenia

folate deficiency intramedullary destruction of blood cell precursors all of the above

8.

Folic acid deficiency and vitamin B12 deficiency both result in megaloblastic anemia with many of the same symptoms and laboratory test results. In the differential diagnosis of the two, which clinical symptom or laboratory test result is unique to the vitamin B12 deficiency associated with pernicious anemia, and can be used to distinguish it from folate deficiency? [Hint] neurologic manifestations macrocytosis pallor, weakness, light-headedness, and shortness of breath slight jaundice

9.

An abnormal Schilling test, Part I, would indicate which of the following? [Hint] dietary B12 deficiency folic acid deficiency pernicious anemia vitamin B6 deficiency

10 .

If an individual went on a starvation diet, which of the following would develop the earliest? [Hint] a deficiency in folate a deficiency in B12 a deficiency in iron a deficiency in B6

Hypoproliferative Anemias Level I Multiple Choice 1. The bone marrow in pure red cell aplasia can be best described as: [Hint] erythroid hypoplasia erythroid hyperplasia hypocellular hypercellular

2.

THe hemoglobin concentration in aplastic anemia is usually: [Hint] less than 3 g/dL less than 5 g/dL less than 7 g/dL less than 12g/dL

3.

The development of aplastic anemia has been associated with exposure to: [Hint] radiation

UV light from the sun air pollution parasites

4.

Improvements in patients with aplastic anemia who are treated with antilymphocyte globulin suggest that the pathophysiology of aplastic anemia may be related to: [Hint] defective bone marrow microenvironment abnormalities in growth factors an autoimmune disorder immunologic suppression of hematopoiesis During the 20th century, the incidence of aplastic anemia appeared to: [Hint] Increase Decrease Remain the same Unable to determine

5.

6.

Erythrocyte morphology in aplastic anemia may be: [Hint] spherocytic, hyperchromic normochromic, macrocytic hypochromic, microcytic hypochromic, macrocytic

7.

Lymphocytes in neutropenic patients with aplastic anemia usually demonstrate: [Hint] a relative increase an absolute increase a relative decrease morphologic abnormalities

8.

A deficiency in EPO is associated with the anemia associated with: [Hint] congenital aplastic anemia aplastic anemia renal disease liver disease

9.

The peripheral blood in patients with aplastic anemia typically reveals: [Hint] pancytopenia bicytopenia granulocytopenia only

anemia only

10 .

Acceptable way(s) to assess the concentration of reticulocytes in patients with aplastic anemia include: [Hint] relative reticulocyte count uncorrected reticulocyte count absolute reticulocyte observation of the smear Hypoproliferative Anemias Level II Multiple Choice 1. Peripheral blood pancytopenia with severe anemia is observed in a 45-year-old female who was treated four years prior for breast cancer. Her bone marrow contains large numbers of neoplastic cells not of hematopoietic origin. What is most probably the cause of her pancytopenia? [Hint] myelophthisic anemia aplastic anemia myeloproliferative disorder pure red cell aplasia A 14-year-old with 7 g/dL hemoglobin has a platelet count of 19 X 109/L and a neutrophil count of 0.4 X 109/L. He has no significant medical history. What test do you think should be performed next? [Hint] a reticulocyte count a bone marrow exam to rule out aplastic anemia iron and TIBC Schilling test

2.

3.

A bone marrow exam was performed on the 14-year-old patient in the previous case, and a diagnosis of idiopathic aplastic anemia was ultimately made. What do you think the results of the bone marrow biopsy revealed? [Hint] a hypercellular marrow consisting of normal hematopoietic precursors a hypercellular marrow with megaloblastic changes a selective decrease in erythroid precursors a hypocellular marrow with decreases in all cell lines

4.

Patients on renal dialysis are usually treated with EPO. Which of the following is a rationale for this therapy? [Hint] to prevent the development of iron deficiency to improve kidney function to compensate for lack of natural EPO production to stimulate liver production of EPO

5.

Constitutional forms of pure red cell aplasia include: [Hint] Diamond-Blackfan syndrome Fanconis anemia transient erythroblastopenia of childhood

congenital dyserythropoietic anemia

6.

A 10-month-old patient has both decreased hemoglobin and reticulocyte count. Leukocytes and platelets are normal. The bone marrow is normocellular with marked erythroid hypoplasia. These results are consistent with a diagnosis of : [Hint] congenital dyserythropoietic anemia Fanconis anemia Diamond-Blackfan syndrome transient erythroblastopenia of childhood

7.

A 10-year-old patient seen in the sickle cell disease clinic has a sudden dramatic decrease in her hemoglobin concentration. She and other family members had recently been ill with a presumed viral infection. A possible explanation for the decrease in her hemoglobin is: [Hint] Her sickle cell disease has worsened. She has acquired a second disorder. She is having a hemolytic episode due to medication. She is experiencing an aplastic crisis.

8.

An anemic patient has many burr cells and schistocytes on the peripheral smear. These results are consistent with which type of hypoproliferative anemia? [Hint] aplastic anemia anemia associated with endocrine disorders pure red cell aplasia anemia of renal disease

9.

Some individuals, not all, develop aplastic anemia after exposure to certain chemicals or drugs. Why might some persons be more susceptible to this condition? [Hint] They may have decreased numbers of suppressor lymphocytes. They may have been exposed to infectious agents. They may have genetic or acquired defects in drug metabolism. They may have fewer numbers of stem cells. A 5-year-old patient with Fanconis anemia with several unaffected siblings would be a possible candidate for which mode of therapy? [Hint] stem cell transplant immunosuppressive chemotherapy none

10 .

Introduction to Hemolytic Anemia Level I Multiple Choice 1 . Which of the following is characteristic of intravascular hemolysis? [Hint] increased haptoglobin

hemosidinuria normal IRF positive AHG test

2 .

Which of the following tests is helpful in evaluating production of erythrocytes in a hemolytic anemia? [Hint] serum bilirubin haptoglobin level methemoglobin concentration reticulocyte count

3 .

A patient with sickle cell disease has an active hemolytic process but his hemoglobin is stable at 11 g/dL. This is an example of: [Hint] compensated hemolytic disease an error in diagnosis a laboratory error in determining hemoglobin extravascular hemolysis

4 .

This clinical finding is associated with hemolytic disease: [Hint] kidney stones gallstones stomach ulcers gastrointestinal parasites

5 .

A patient with spherocytic hemolytic anemia has an increased serum bilirubin, but haptoglobin is normal and there is no hemosidinuria or hemoglobinuria. This indicates: [Hint] intravascular hemolysis extravascular hemolysis compensated hemolytic disease an intrinsic erythrocytic defect

Introduction to Hemolytic Anemia Level II Multiple Choice Top of Form 1 . Which of the following reticulocyte counts is typical of a hemolytic anemia? [Hint] decreased IRF 1% 500 x 109/L 10 x 109//L

2 .

A patient with hereditary spherocytosis who previously had a stable hemoglobin of 11 g/dL now has a decreasing hemoglobin (9 g/dL). This is an example of: [Hint]

aplastic anemia complicating autoimmune hemolytic anemia uncompensated hemolytic disease compensated hemolytic disease

3 .

A patient has sudden onset of intravascular hemolytic anemia due to a snake bite. Which of the following is a characteristic clinical finding of this type of hemolysis? [Hint] expansion of bone marrow abnormalities of the skeleton extramedullary masses of erythropoietic tissue red urine A patients hemoglobin is rapidly dropping. The blood smear reveals many schistocytes. The urine contains hemoglobin. Which of the following is the most likely cause of this anemia? [Hint] microangiopathic hemolytic anemia hereditary spherocytosis autoimmune hemolytic anemia red cell enzyme deficiency

4 .

5 .

A patient is suspected of having an immune hemolytic anemia. What laboratory test would be helpful in confirming this suspicion? [Hint] AHG reticulocyte count reticulocyte count serum bilirubin haptoglobin

Bottom of Form Hemolytic Anemia: Membrane Defects Level I Multiple Choice 1. The most common membrane defect in common hereditary elliptocytosis is: [Hint] a spectrin deficiency a defective spectrin a C8bp defect the presence of a mutant spectrin

2.

The definitive test for paroxysmal nocturnal hemoglobinuria is the: [Hint] sucrose hemolysis test osmotic fragility test autohemolysis test Ham test

3.

All of the following erythrocyte membrane disorders are associated with abnormal membrane permeability except: [Hint] PNH hereditary xerocytosis hereditary stomatocytosis hereditary spherocytosis

4.

Patients with common Hereditary elliptocytosis generally are asymptomatic. [Hint] true false

5.

Which of the following is most true about the etiology of paroxysmal nocturnal hemoglobinuria (PNH)? [Hint] It is inherited as an autosomal recessive trait. It is an acquired hemolytic anemia. It is inherited as a sex-linked trait. It is inherited as an autosomal dominant trait.

6.

The morphologic hallmark of hereditary spherocytosis on the peripheral smear is: [Hint] spherocytes polychromasia ovalocytes excessive poikilocytosis

7.

A patient has hemoglobin of 99 g/L and a hematocrit of 27 %. Calculate the MCHC and determine which of the following erythrocyte disorders would show an MCHC similar to the one you calculated: [Hint] hereditary stomatocytosis hereditary elliptocytosis hereditary spherocytosis PNH

8.

An erythrocyte membrane disorder that results in increased sensitivity to complement lysis is called: [Hint] paroxysmal cold hemoglobinuria (PCH) hereditary spherocytosis hereditary stomatocytosis paroxysmal nocturnal hemoglobinuria (PNH)

9.

Erythrocytes that are osmotically fragile can take on more water than normal erythrocytes. [Hint] true false

10 .

The screening test that is useful in identifying erythrocytes that are abnormally sensitive to complement lysis is: [Hint] osmotic fragility test

sucrose hemolysis test autohemolysis test Ham test

Hemolytic Anemia: Membrane Defects Level II Multiple Choice 1. Which of the following combinations is characteristically seen in hereditary spherocytosis? [Hint] Decreased surface-area-to-volume ratio, increased MCV, and decreased MCHC Increased surface-area-to-volume ratio, decreased MCV, and increased MCHC Decreased surface-area-to-volume ratio, increased osmotic fragility, and increased MCHC Increased surface-area-to-volume ratio, decreased osmotic fragility, and decreased MCHC

2.

Which of the following results from the Ham test would indicate a diagnosis of PNH? [Hint] patient red cells + .2N HCL + normal serum = no hemolysis patient red cells + .2N HCL + normal serum = 3+ hemolysis patient red cells + normal serum = 1+ hemolysis patient red cells + .2N HCL + heat inactivated serum = 1+ hemolysis

3.

What does HE (4.1) denote? [Hint] hereditary elliptocytosis with 4 defects on band 1 hereditary spherocytosis with a defective E gene at position 4.1 hereditary elliptocytosis with absence of protein 4.1 hereditary pyropoikilocytosis with defective protein 4.1

4.

Which of the following can be seen in hereditary elliptocytosis? [Hint] greater than 25 % elliptocytes on the peripheral smear an increased osmotic fragility curve autosomal recessive inheritance pattern positive sucrose hemolysis test

5.

Which of the following describes the role of decay accelerating factor (DAF) on erythrocytes? [Hint] DAF inactivates C8 binding protein. DAF prevents the amplification of C3/C5 convertase activity. DAF interferes with the terminal stages of complement activation that causes erythrocyte lyisis. DAF amplifies C3/C5 convertase activity.

6.

Which of the following is a characteristic of hereditary pyropoikilocytosis (HPP)? [Hint] RBC fragments and micropoikilocytes present Erythrocytes have decreased thermal sensitivity.

erythrocytes that are osmotically normal abnormal Ham test

7.

Which of the following is characteristically seen in hereditary stomatocytosis ? [Hint] Positive Ham test decreased MCV and increased MCHC increased osmotic fragility Reticulocytopenia

8.

Which of the following is characteristically seen in hereditary xerocytosis (HX)? [Hint] decreased MCHC increased osmotic fragility dehydrated red blood cells positive Ham test

9.

Which of the following protein defects or deficiencies is associated with a horizontal interaction defect? [Hint] protein 4.2 ankyrin adducin band 3

10 .

Which of the following combinations best describes paroxysmal nocturnal hemoglobinuria (PNH)? [Hint] pancytopenia, positive sucrose hemolysis, positive Ham test, and decreased LAP score leukopenia, thrombocytopenia, negative Ham test, and decreased LAP score sex-linked inheritance, positive Ham test, and decreased LAP score acquired hemolytic anemia, positive Ham test, and increased LAP score

Hemolytic Anemia: Metabolic Defects Level I Multiple Choice Top of Form 1. Which metabolic pathway is responsible for keeping oxidants in check in the erythrocyte? [Hint] HMP shunt EM Krebs cycle 2,3-DPG

2.

This enzyme deficiency does not usually manifest itself by hemolysis unless there is increased oxidant stress on the erythrocyte: [Hint] PK

TPI G6PD glutathione synthetase

3.

Bite cells are associated with a deficiency in this enzyme. [Hint] PK G6PD TPI glutathione peroxidase

4.

Screening tests for G6PD deficiency include: [Hint] spectrophotometric quantitation of G6PD spectrophotometric quantitation of PK dye reduction test incubation with PEP and ADP

5.

The etiology of G6PD is: [Hint] autosomal recessive inheritance acquired stem cell defect autosomal dominate inheritance sex-linked inheritance

6.

Favism is associated with a deficiency in this enzyme: PK G6PD TPI glutathione synthetase

7.

A deficiency in this enzyme will lead to a loss of red cell energy in the form of ATP: [Hint] PK G6PD TPI glutathione synthetase

8.

In which group is symptomatic G6PD found most frequently? [Hint] Caucasians Asians Females Males

9.

This most commonly precipitates a hemolytic attack in individuals with G6PD deficiency: [Hint] drug oxidants cancer hyperglycemia high altitude

10 .

Enzymes function in the mature red cell to: [Hint] break down hemoglobin destroy invading microorganisms supply energy and reduce oxidants synthesize hemoglobin

Bottom of Form Hemolytic Anemia: Metabolic Defects Level II Multiple Choice Top of Form 1. A 20-year-old black man was suspected of having G6PD deficiency when he experienced hemolytic anemia after administration of primaquine. An erythrocyte G6PD analysis, performed on blood taken four days after symptoms appeared, was normal. A reticulocyte count revealed 20% reticulocytes at this time. These results suggest:[Hint] The patient does not have G6PD deficiency but may have PK deficiency. Leukocytes may be contaminating the sample and causing a false negative result. Another G6PD test should be done in several months. The patient probably has the G6PD Mediterranean variant.

2.

Why should testing for enzyme deficiencies be delayed immediately after a hemolytic attack? [Hint] Reticulocytes may have normal enzyme levels. Older cells have more normal enzyme levels. There are not enough erythrocytes present to do the testing. It is better to do the testing on the entire family at the same time.

3.

Echinocytes are associated with this disorder: [Hint] G6PD deficiency PK deficiency 2,3 DPG deficiency abnormal G6PD enzyme

4.

What metabolic process is inhibited in a deficiency of red cell PK? [Hint] reduction of peroxides production of ATP from glycolysis production of 2,3 DPG from glucose

formation of GSH

5.

Which of the following statements is true concerning G6PD deficiency? [Hint] Affected patients will have chronic hemolytic anemia from birth. Carrier females do not have enzyme-deficient cells. Most patients do not have symptoms associated with the deficiency. Infants are commonly affected with hemolysis.

6.

If hemoglobin is oxidized, it denatures in the form of: [Hint] basophilic stippling Dohle bodies Howell-Jolly bodies Heinz bodies

7.

What test should be done on blood if bite cells are found on a Romanowsky-stained blood smear? [Hint] Heinz body stain reticulocyte stain iron stain reticulocyte count

8.

This important erythrocyte buffer prevents oxidant damage to hemoglobin and is dependent on G6PD for maintaining adequate levels: [Hint] 2,3 DPG glutathione H2O2 glutathione reductase

9.

G6PD A+ and G6PD A- can be differentiated by this laboratory test: [Hint] electrophoresis measurement of formation of ATP when blood is incubated with PEP and ADP spectrophotometric analysis of the enzyme CBC

10 .

These G6PD isoenzymes have normal enzyme activity: [Hint] G6PD A+ and G6PD Mediterranean G6PD A+ and G6PD Canton G6PD Canton and G6PD Mediterranean G6PD A+ and G6PD B

Bottom of Form 1 What is the most common pattern of the DAT in cases of WAIHA ( +,

agglutination; =, no agglutination)?

Poly specific AHG [Hint] + + + + 2 .

anti-IgG

anti-C

= = + +

+ = + =

The specificity of the antibody in PCH is most commonly directed against what erythrocyte antigen? [Hint] ABO Rh I P

3 .

ABO HDN is characterized by which of the following erythrocytes on the peripheral blood smear? [Hint] target cells acanthocytes spherocytes macrocytes

4 .

The specificity of the antibody in cold agglutinin syndrome is directed against what erythrocyte antigen? [Hint] ABO I P Rh

5 .

Extravascular hemolysis is associated with all of the following conditions EXCEPT: [Hint] Rh HDN delayed transfusion reaction WAIHA acute hemolytic transfusion reaction

6 .

The antibodies most frequently linked to delayed hemolytic transfusion reactions belong to what system? [Hint] ABO Duffy Kidd

Rh

7 .

Following birth, the greatest risk to a newborn with Rh HDN is due to: [Hint] decreased hemoglobin decreased hematopoieses increased alloantibody concentration increased bilirubin levels

8 .

What DAT profile is most common in patients with cold agglutinin syndrome? Poly anit-IgG +++ ++= +=+ +== anti-C3 [Hint]

9 .

The characteristic cell associated with Rh HDN [Hint] nucleated RBC macrocyte schistocytes target cell

1 0 .

Antibodies associated with extravascular hemolysis are most commonly of what immunoglobulin class? [Hint] IgA IgD IgG IgM

Hemolytic Anemia: Nonimmune Defects Level I Multiple Choice 1. Spur cell anemia is associated with which of the following conditions? [Hint] thermal burns bartonellosis cirrhosis abetalipoproteinemia

2.

A patient with MAHA and abnormal coagulation tests, including a prolonged protime and APTT, and increased levels of FDP would most likely have which of the following underlying conditions? [Hint] TTP DIC HUS

LCAT deficiency

3.

A person diagnosed with MAHA due to HUS would most likely be in which age group? [Hint] <10 years old 11-25 26-40 >40

4.

Identify several organisms that may cause erythrocyte hemolysis. [Hint] Shigella dysenteriae Type I Salmonella typhi serotype typhi Campylobacter jejuni , subspecies jejuni Escherichia coli 0157:H7

5.

The underlying cause of RBC damage in patients with burns is: [Hint] damage to spectrin loss of glycoproteins thermal scarring formation of spicules

6.

All of the following have been identified as causes of secondary HUS in adults except: [Hint] complications of pregnancy systemic lupus erythematosus cancer kidney stones

7.

Which of the following is found in patients with TTP but not in patients with HUS? [Hint] thrombocytopenia fever renal failure schistocytes

8.

In a patient with severe MAHA and a hemoglobin of 12 g/dl, which of the following parameters would most likely be significantly decreased? [Hint] MCHC MCV MCH reticulocyte count

9.

The primary initiating factor in MAHA is: [Hint] consumption of coagulation factors

abnormal erythrocyte membrane formation damage to capillary endothelial cells intravascular hemolysis

10 .

Anemia without the presence of schistocytes would be seen in which of the following: [Hint] HUS TTP malaria DIC Hemolytic Anemia: Nonimmune Defects Level II Multiple Choice 1. A type of hemolytic process characterized by fragmentation of RBCs due to excessive force as they circulate through microcapillaries is known as: [Hint] March hemoglobinuria HUS TTP DIC

2.

The most likely underlying cause of TTP is: [Hint] presence of a vWF anomaly lack of vWF cleaving protease defect in protaglandid I2 synthesis depressed fibrinolysis

3.

The underlying cause of organ damage in HUS is due to: [Hint] thrombosis of the microcirculation intravascular hemolysis abnormal clot lysis coagulation abnormalities

4.

When there is marked fragmentation of the RBCs due to intravascular damage, what RBC parameter is affected most? [Hint] MCV MCHC PDW G6PD

5.

What would you expect the result of the immature reticulocyte fraction (IRF) to be in MAHA? [Hint] decreased normal

increased impossible to tell

6.

A patient developed bacterial sepsis after multiple injuries in a farm accident. He had a sudden drop in hemoglobin and follow up studies showed the following: PT 25 sec. APTT 55 sec. FDP increased fibrinogen 100 mg/dl platelet count 34, 000 What is the most likely underlying cause of these results? [Hint] TTP delayed transfusion reaction DIC acute hemolytic transfusion reaction

7.

A mother brought her 5-year-old boy to the physician because of decreased urinary output despite normal fluid intake. Laboratory results show decreased hemoglobin [90 g/L], decreased platelet count, normal coagulation values and the presence of large numbers of schistocytes on the peripheral blood smear. She denies any episodes of bloody diarrhea in the boy. What is the most likely clinical condition? [Hint] DIC (D-) HUS TTP LCAT deficiency

8.

What combination of test results would help to distinguish DIC from TTP as a cause of schistocytosis? PT &APTT Fibrinogen FDP [Hint] prolonged decreased increased prolonged normal normal normal increased normal decreased increased decreased

9.

The purpose of using plasma exchange to treat TTP is: [Hint] removal of fibrinogen removal of platelets addition of coagulation factors replacement of vWF cleaving protease

10 .

Which of the following hemolytic anemias is not associated with schistocytes? [Hint] TTP March hemoglobinuria DIC extensive thermal burns

Nonmalignant Granulocyte and Monocyte Disorders

Level I Multiple Choice 1. A patient has a white count of 18 X109/L. Which of the following is true? [Hint] normal for both an adult and a newborn high for both an adult and a newborn normal for an adult but high for a newborn high for an adult but normal for an infant

2.

Which patient demographic normally has more lymphocytes than neutrophils? [Hint] African-American adults children newborn babies women

3.

Which of the following typically presents with the highest white counts, the most immature cells, and a low LAP? [Hint] CML gout trauma chronic inflammation

4.

Which of the following describes a peripheral leukoerythroblastic reaction? [Hint] increased monocytes immature granulocytes and nucleated erythrocytes increased eosinophils and basophils increased specific or pink granules in neutrophils

5.

Pelger-Huet cells may be confused with which cell type? [Hint] bands monocytes eosinophils basophils

6.

What most often causes cytoplasmic vacuoles in neutrophils? [Hint] chemotaxis lipids phagocytosis inadequate production of granules

7.

Dhle bodies are most often associated with which other feature? [Hint] eosinophilia

monocytosis hypersegmentation toxic granulation

8.

Which of the following can be produced as a storage artifact of blood? [Hint] toxic granulation Dhle bodies vacuoles Chediak-Higashi inclusion

9.

Monocytosis and basophilia are most often associated with which of the following? [Hint] viral infections drugs parasitic infections certain leukemias and myeloproliferative conditions

10 .

Which leukocyte is seen in the smallest concentration in normal peripheral blood? [Hint] lymphocyte monocyte eosinophil basophil

Nonmalignant Lymphocyte Disorders Level I Multiple Choice 1. Which of the following leukocyte differentials typically is found in EBV infections? [Hint] neutrophils 76%, lymphocytes 14%, monocytes 8%, eosinophils 2%, basophils 0% neutrophils 88%, lymphocytes 6%, monocytes 4%, eosinophils 1%, basophils 1% neutrophils 60%, lymphocytes 32%, monocytes 3%, eosinophils 4%, basophils 1% neutrophils 42%, lymphocytes 52%, monocytes 4%, eosinophils 1, basophils 1% The best description of the patients leukocyte count in question #1 is: [Hint] neutrophilia eosinophilia monocytosis lymphocytosis Reactive lymphocytes that are 10-12 in size, with blue cytoplasm and an irregular cytoplasmic membrane, are typical of: [Hint] normal resting T-lymphocytes T-lymphocytes reacting to Epstein-Barr virus

2.

3.

normal resting B-lymphocytes lymphocytes reacting to Bordetella pertussis

4.

The lymphocytes found in infectious lymphocytosis are similar to those found in: [Hint] whooping cough infectious mononucleosis viral infections leukemia

5.

A positive heterophil antibody test is typically found in: [Hint] SCIDS infectious lymphocytosis Bordetella pertussis infectious mononucleosis

6.

Which of the following symptoms are typical of a reactive lymphocytosis? [Hint] jaundice and microcephaly hepatomegaly and mental retardation thrombocytopenia and hemolytic anemia pharyngitis and lymphadenopathy

7.

The most common viral infection complicating organ transplants is: [Hint] CMV EBV Coxsackie virus HIV

8.

Which of the following conditions is associated with lymphocytopenia? [Hint] whooping cough infectious mononucleosis infectious lymphocytosis malnutrition

9.

Which of the following is typically found in HIV-1 infection? [Hint] decreased T-lymphocytes leukocytosis monocytosis neutrophilia

10 .

A patient infected with HIV who is on zidovine typically shows: [Hint]

macrocytic anemia microcytic anemia leukocytosis hypersegmented PMNs

11 .

The confirmatory test for infectious mononucleosis is: [Hint] CBC heterophil antibody test EBV viral load HIV viral DNA

Myeloproliferative Disorders Level I Multiple Choice 1. The most prominent cell line found in myeloid metaplasia with myelofibrosis is the: [Hint] erythroid myeloid megakaryocyte fibroblast

2.

The cytogenetic translocation between chromosome 9 and 22 that results in the Philadelphia chromosome is diagnostic for: [Hint] MPD PV CML CMML

3.

Blast crisis according to the WHO classification system is recognized by: [Hint] >20% blasts in bone marrow thrombocytopenia and anemia overt leukocytosis with <10% blasts in bone marrow lymphadenopathy and bone pain

4.

A CML patient whose disease accelerates through transition phase will have a clonal increase of __________ blasts in the bone marrow: [Hint] myeloid lymphoid erythroid any type of

5.

In a leukemoid reaction, the: [Hint] LAP score is elevated and Ph-chromosome is absent

LAP score is decreased and Ph-chromosome is absent LAP score is normal and Ph-chromosome is present LAP score is elevated and Ph-chromosome is present

6.

A patient with MMM could likely have _________ on the peripheral blood film: [Hint] thrombocytosis with bizarre platelets an extreme leukocytosis, WBC >250 x 109/L a reactive lymphocytosis a left shift with many promyelocytes and blasts

7.

In CML, thrombocytopenia is found during: [Hint] chronic phase accelerated phase blast crisis phase not found, CML patients have a thrombocytosis

8.

Which cells characteristically show a normal karyotype in MMM? [Hint] erythroblasts myeloblasts megakaryoblasts fibroblasts Leukocytosis with occasional immature granulocytes and sustained thrombocytosis with >1,000,000 x 10 9/L is found in: [Hint] chronic myelogenous leukemia myelofibrosis with myeloid metaplasia reactive leukocytosis essential thrombocytosis A 53-year-old male had a CBC during a routine physical exam. The WBC = 47 x 109/L with a differential of 16% segs, 12% bands, 18% metamyelocytes, 26%myelocytes, 6% promyelocytes, 2% blasts, 10% lymphocytes, 4% monocytes, 4% basophils, and 2% eosinophils. Platelets = 400 x 109/L and RBC = 2.8 x 1012/L. He most likely has:[Hint] AML CML PV ET

9.

10 .

Acute Myelogenous Leukemias Level I Multiple Choice 1. A differential report notes the presence of more than 30% blasts. This number supports the diagnosis of: [Hint] ALL - most probably childhood ALL AML - most probably M1

AL - of uncertain origin either AL or CML

2.

The presence of myeloblasts with little to no maturation is commonly seen in: [Hint] AML - M1 AML - M2 AML - M5 AML - M6

3.

Most acute leukemias demonstrate a M:E ratio of: [Hint] 1:1 1:5 10:1 4:1

4.

A large cell with an immature nucleus containing multiple prominent nucleoli and few azurophilic granules is a: [Hint] type 1 myeloblast type 2 myeloblast promyelocyte promyelocyte with hypogranulation

5.

A positive result with the chloro AS-D acetate esterase stain supports a diagnosis of: [Hint] M4 M5 M6 M7

6.

PAS stain differs from the traditional stains used in cytochemistry in that PAS: [Hint] requires a fresh, non-anticoagulated specimen is reported as positive or negative in different cell lines produces characteristic patterns of positivity in different cell lines is better used in conjunction with monoclonal antibody testing

7.

Auer rods have been determined to be: [Hint] composed of histamine positive for proteolytic esterases diagnostic for AML present in high numbers in adolescent AML

8.

A bone marrow contains many blast cells that stain positively for alpha naphthyl acetate. This reaction indicates that these cells are most likely of what lineage? [Hint]

erythroblastic lymphoblastic monoblastic myeloblastic

9.

When Auer rods (bodies) are found in blasts of a case of acute leukemia, the leukemia is most probably: [Hint] undifferentiated leukemia B-lymphocytic leukemia T-lymphocytic leukemia myelocytic leukemia

10 .

Circulating micromegakaryocytes may be found in which of the following acute leukemias? [Hint] M4 M5 M6 M7

Acute Lymphoblastic Leukemias Level I Multiple Choice 1. In acute lymphoblastic leukemia, the blast cells can be positive for: [Hint] Sudan black B myeloperoxidase PAS metachromasia

2.

ALL1 is often described as: [Hint] adult onset ALL childhood ALL Szary syndrome mycosis fungoides

3.

The lymph nodes most commonly seen in ALL are described as: [Hint] large and painful small and shotty large and hard small and painful

4.

The presence of a heterogeneous collection of lymphoblasts in the marrow is usually a sign of: [Hint] L1

L2 L3 mixed lineage leukemia

5.

A disease characterized by a neoplastic proliferation of undifferentiated lymphoblasts in the bone marrow is: [Hint] AML CML ALL CLL A patient with a WBC of 15 x 109/L and 80% immature lymphoid cells in the peripheral blood is most likely: [Hint] AML ALL CML CLL

6.

7.

ALL is most commonly found in this age group: [Hint] elderly children between 2-5 years of age adults in the fifth decade of life teenagers

8.

ALL can be differentiated from AML in the laboratory by: [Hint] the platelet count the white blood cell count identifying the blast cell lineage degree of anemia

9.

Lymphoblasts in ALL are typically: [Hint] Sudan black B and MPO negative and PAS positive PAS, Sudan black B and MPO positive Negative with all cytochemical stains Tdt negative but MPO positive

10 .

A blast cell population from a leukemia patient was analyzed. It had cell markers of both lymphoblasts and myeloblasts. How can this be explained? [Hint] The patient has both AML and ALL. It is a biphenotypic leukemia. The laboratory stains are not working properly.

Chemotherapy is interfering with the cytochemical stains. Acute Lymphoblastic Leukemias Level II Multiple Choice 1. A peripheral white blood cell count of 9.4x109/L would: [Hint] rule out a diagnosis of ALL be of substantive weight in the diagnosis of L3 neither confirm nor deny a potential diagnosis of ALL confirm a diagnosis of childhood ALL while excluding a diagnosis of L3

2.

CD2 is positive in: [Hint] B cells precursor B cells T cells precursor T cells

3.

Surface immunoglobulins are most often found on: [Hint] pre-T cells NK cells pre-B cells B cells

4.

Another name for the common ALL antigen (cALLA) is: [Hint] CD2 CD10 CD19 CD20

5.

The flow cytometry results for a patient with a suspected lymphoproliferative disorder are: CD2 CD10 CD19 CD20 negative negative positive positive

Which of the following conditions would be supported? [Hint] childhood ALL early B cell ALL B cell ALL T cell ALL

6.

Abnormalities in chromosome 8 are often seen in cells that are also positive for: [Hint] CD2 CD10

Tdt sIg

7.

A Patient has the t(9;22) aberration, but the leukemic blasts are MPO and SBB negative. What is the most likely explanation for these results? [Hint] The patient has CML. The patient has Ph+ AML. The patient has Ph+ ALL. The cytochemical stains are not working properly.

8.

Molecular analysis of a population of leukemic blasts reveals rearrangement of the immunoglobulin gene. This is positive evidence for: [Hint] T-cell ALL undifferentiated AL mixed lineage AL B-cell ALL

9.

A patient has ALL. The blasts are large with abundant cytoplasm. Some blasts have vacuoles. Molecular analysis reveals the cMYC gene rearrangement. What immunologic markers would you expect on these blasts? [Hint] CD19, CD22, CD10 CD3, CD5, CD7 CD34, CD33, CD13 CD19, CD22, CD4

10 .

What is the most probable FAB subgroup of the leukemia in the patient in question #9? [Hint] M0 L1 L2 L3 Lymphoid Malignancies Level I Multiple Choice 1. A diagnosis of hairy cell leukemia is established in a 50-year-old man. Which of the following is the most likely CBC finding at presentation? [Hint] lymphocytosis > 10 x 109/L monocytosis neutrophilia pancytopenia

2.

Which of the following phenotypes is characteristic of hairy cell leukemia? [Hint] CD19+, CD5-, CD11c+, CD25+ CD19+, CD5-, CD11c+, CD25-

CD19+, CD5+, CD11c+, CD25+ CD19+, CD5+, CD11c+, CD25-

3.

A patient presents with widespread lymphadenopathy involving multiple peripheral nodes, a tumorous mass involving the gastrointestinal tract, and malignant cells in the peripheral blood. Which of the following is the most likely diagnosis? [Hint] non-Hodgkins lymphoma Hodgkins disease chronic lymphocytic leukemia multiple myeloma

4.

Which of the following findings indicates the presence of follicular lymphoma rather than reactive follicle germinal centers? [Hint] a homogeneous population of lymphocytes absence of bcl-2 protein abundant apoptosis numerous mitotic figures

5.

Which of the following is the correct definition of a tingible body macrophage? [Hint] a benign macrophage ingesting apoptotic lymphocytes a benign macrophage ingesting organisms a dark staining neoplastic macrophage a neoplastic lymphocyte that resembles a macrophage

6.

EBV plays a role in the pathogenesis of which of the following types of lymphoma? [Hint] Burkitts lymphoma small lymphocytic lymphoma follicular lymphoma diffuse large cell lymphoma Which of the following karyotypes is the most likely finding in Burkitts lymphoma? [Hint] t(8;14) t(14;18) t(9;22) t(15;17)

7.

8.

A lymph node biopsy revealed an abnormal infiltrate containing Reed-Sternberg cells. Which of the following is the most likely diagnosis? [Hint] Hodgkins lymphoma non-Hodgkins lymphoma multiple myeloma acute lymphoblastic leukemia

9.

Which of the following is/are involved in the classification of plasma cell neoplasms? [Hint] distribution of disease the type of immunosecretory protein the appearance of the malignant cells all of the above

10 .

Which of the following is the correct definition of Bence Jones proteins? [Hint] immunoglobulin light chains in the urine immunoglobulin light chains in the serum monoclonal serum immunoglobulin monoclonal urine immunoglobulin

Lymphoid Malignancies Level II Multiple Choice 1. A lymph node biopsy performed on a patient with a previous diagnosis of low-grade non-Hodgkins lymphoma revealed a diffuse infiltrate of large lymphoid cells, extensive necrosis, and numerous mitotic figures. Which of the following is the most likely diagnosis on the current sample? [Hint] follicular non-Hodgkins lymphoma diffuse large cell non-Hodgkins lymphoma small lymphocytic non-Hodgkins lymphoma Hodgkins disease

2.

Which of the following is the most likely site of presentation of MALT lymphoma? [Hint] bone marrow lymph node peripheral blood stomach

3.

Which of the following statements explains the difficulty that is often encountered in rendering a diagnosis of MALT lymphoma? [Hint] The infiltrate contains a mixture of benign and malignant cells. The neoplastic cells have a variety of appearances. The phenotype is not specific (CD10-, CD5-). all of the above

4.

A 20-year-old male presents with a large mediastinal mass. Biopsy reveals a uniform infiltrate of intermediate-sized, immature lymphoid cells. Flow cytometry reveals the following phenotype: CD3-, CD2+, CD4+, CD8+, CD10+. Which of the following is the most likely diagnosis? [Hint] anaplastic large cell lymphoma lymphoblastic lymphoma peripheral T/NK-cell lymphoma Szary syndrome

5.

Which of the following Hodgkin variant cells is characteristic of the nodular sclerosis subtype? [Hint]

lacunar popcorn mononuclear Reed-Sternberg

6.

A 50-year-old female presented with visual impairment, headaches, dizziness, and deafness. Physical examination revealed lymphadenopathy, hepatomegaly, and splenomegaly. Laboratory studies revealed an increased serum protein and a monoclonal serum protein migrating in the gamma region. What is the most likely class of the immunoglobulin? [Hint] IgG IgE IgM IgD

7.

A 70-year-old patient presents with a fracture of the femur. Tissue removed at the time of bone fixation reveals an infiltrate of monoclonal plasma cells. Radiology studies fail to reveal any other bony lesions, and bilateral posterior iliac crest bone marrow specimens reveal only 1% plasma cells. Which of the following is the most appropriate diagnosis? [Hint] multiple myeloma plasmacytoma monoclonal gammopathy of undetermined significance uncomplicated fracture

8.

A patient presented with a fracture of his femur. Radiology studies revealed multiple lytic bone lesions. Which of the following is the most likely diagnosis? [Hint] Hodgkins lymphoma non-Hodgkins lymphoma multiple myeloma acute lymphoblastic leukemia

9.

Which of the following is the most likely peripheral blood finding in a patient with multiple myeloma? [Hint] blasts plasma cells plasmacytoid lymphocytes Rouleaux formation

10 .

A 71-year-old asymptomatic female was found to have a small monoclonal spike on serum protein electrophoresis. Which of the following findings would confirm a diagnosis of monoclonal gammopathy of undetermined significance (MGUS)? [Hint] Bence Jones proteins 8% plasma cells in the bone marrow lytic bone lesions functional hypogammaglobulinemia

Primary Hemostasis Level I Multiple Choice

1.

When a blood clot forms inappropriately inside a blood vessel, the process is called: [Hint] hemostasis thrombosis blood coagulation hematopoiesis

2.

Which of the following processes is not an element of primary hemostasis? [Hint] platelet aggregation platelet adhesion vasoconstriction formation of fibrin

3.

The primary hemostatic plug consists of: [Hint] aggregated platelets fibrin platelets plus fibrin aggregated leukocytes When comparing the size of normal platelets to the size of normal erythrocytes on a Wrights -stained peripheral blood smear: [Hint] They are equal in size. Platelets are approximately one-third the diameter of the erythrocytes. Platelets are usually three times larger than the erythrocytes. The size of normal platelets varies so much that they cannot be compared.

4.

5.

Which of the following is within a reasonable reference range for platelets? [Hint] 100 x 1012/L 10 x 109/L 1000 x 109/L 200 x 109/L

6.

Vasoconstriction is a function of [Hint] platelets blood vessels erythrocytes hemostatic proteins in the plasma

7.

Platelet aggregation is the process of: [Hint] platelets binding to collagen platelets causing vasoconstriction

platelets binding to one another platelets binding to endothelial cells

8.

Megakaryocytes are: [Hint] cells in the peripheral blood that prevent blood clotting precursor cells to platelets in the bone marrow cells in the tissues to which platelets adhere during primary hemostasis the clump of platelets that forms during primary hemostasis

9.

The primary hemostatic plug is: [Hint] cells in the peripheral blood that prevent blood clotting precursor cells to the platelets in the bone marrow cells in the tissues to which platelets adhere during primary hemostasis the clump of platelets that forms during primary hemostasis

10 .

The events that occur in forming the primary hemostatic plug are: [Hint] platelet adhesion, aggregation, secretion vasoconstriction, fibrin formation, fibrinolysis platelet aggregation and fibrinolysis platelet adhesion and fibrin formation

Secondary Hemostasis and Fibrinolysis Level I Multiple Choice Top of Form 1. The "fibrinogen group" of coagulation proteins includes: [Hint] factors II, VII, IX, and X factors XII, IX, PK, and HK factors I, V, VIII, and XIII factors I, VII, TF, and XIII

2.

Which of the following coagulation factors is NOT a serine protease? [Hint] factor IIa factor XIIa factor XIa factor XIIIa

3.

Which of the following coagulation factors are involved in the "common pathway" of coagulation? [Hint] factors VII, V, II, and I factors X, V, II, and XIII

factors IX, VIII, II, and I factors XI, XI, VIII, and IX

4.

Zymogens are: [Hint] inactive precursors of proteases active serine proteases cofactors in the coagulation cascade activators of contact activation factors

5.

Deficiency of which of the following proteins is associated with a prolonged aPTT, but no evidence of defective hemostasis in vivo? [Hint] factor X factor VII factor XII factor XIII

6.

Which of the following coagulation factors serves as a cofactor in the "contact activation" phase of coagulation? [Hint] factor XII factor XI prekallikrein high molecular weight kininogen

7.

Which of the following coagulation factors constitute the "intrinsic Xase" complex? [Hint] factors XIIa, XIa, Ixa, and VIIIa factors VIIa, TF, and Ca2+ factors IXa, VIIIa, phospholipid, and Ca2+ factors Xa, Va, phospholipid, and Ca2+

8.

The main function of PAI-1 is to: [Hint] inhibit plasmin inhibit plasminogen activators inhibit factor XIIIa inhibit the XIIa, XIa, and kallikein

9.

The activity of which of the following protease inhibitors is accelerated by heparin? [Hint] antithrombin 2-antiplasmin 2macroglobulin 1antitrypsin

10 .

Protein C and protein S function as: [Hint]

procoagulant cofactors inhibitors of fibrinolysis inhibitors of contact activation inhibitors of coagulation Bottom of Form Thrombophilia Level I Multiple Choice 1. An increased tendency to form venous thromboemboli is called: [Hint] procoagulant hyperreactive coagulation thrombophilia hemophilia

2.

A thromboembolus is: [Hint] formation of a mass of platelets within a vessel formation of a fibrin mass within a vessel formation of a platelet/fibrin mass within a vessel a piece of thrombotic material released from a thrombus into the circulatory system

3.

All of the following are considered risk factors for the formation of venous thrombi EXCEPT: [Hint] atherosclerotic plaques deficiency of protein C or protein S elevated prothrombin levels elevated plasminogen activator inhibitor (PAI-1) levels

4.

Which of the following statements concerning the laboratory diagnosis of DVT is correct? [Hint] The laboratory diagnosis depends on unique assays specific for venous thrombosis. Radiologic procedures to visualize the thrombus may aid in the identification of thrombi. The quantitative D-dimer assay is specific for DVT. The TAT assay (thrombin-antithrombin complex) is specific for DVT.

5.

Heparin catalyzes the anticoagulant action of which plasma coagulation inhibitor? [Hint] antithrombin protein C heparin cofactor II both a and c

6.

Protein S deficiency predisposes to thrombosis because: [Hint] There is diminished inhibition of contact activation.

There is diminished activation of antithrombin. There is inadequate protein C inactivation of Va and VIIIa. There is diminished activation of plasminogen

7.

Antithrombin deficiency as a cause of inherited thrombophilia may be due to: [Hint] synthesis of reduced amounts of antithrombin (a quantitative deficiency) production of a normal quantity of a nonfunctional protein production of excess heparan sulfate by endothelial cells both a and b

8.

Activated Protein C Resistance (APCR) contributes to a thrombophilic tendency by: [Hint] decreased activation of protein C by thrombin/thrombomodulin decreased protein S cofactor activity for protein C decreased destruction of factor Va by activated protein C decreased destruction of factor VIIIa by activated protein C

9.

The major advantage of using low-molecular weight heparin vs. unfractionated heparin is: [Hint] LMWH has a more predictable dose-response profile. LMWH does not bind heparin cofactor II. LMWH does not usually require routine laboratory monitoring. both a and c

10 .

The best test for monitoring oral anticoagulant therapy is: [Hint] Prothrombin Time (PT) International Normalized Ratio (INR) Activated Partial Thromboplastin Time (APTT) Thrombin Time (TT)

1 .

The hematology laboratory informs you that the hematocrit on patient Z is 62% (0.62 L/L). For the proper anticoagulation of a 5.0 mL sample of patient Z's blood, how much 3.2% sodium citrate should be used? [Hint] 0.18 mL 0.35 mL 0.43 mL 0.62 mL

2 .

The laboratory test used to monitor heparin therapy is: [Hint] factor Xa inhibition assay bleeding time prothrombin time

reptilase time

3 .

Which is the specimen of choice for the fibrinogen determination? [Hint] platelet-rich plasma, citrated platelet-poor plasma, citrated serum plasma, heparinized

4 .

Which of the following is NOT identified by the FDP test? [Hint] fragment D fragment X fragment E D-dimer

5 .

Given the following data, calculate the INR: Patient's PT 18.5 seconds Mean normal PT 11.5 seconds ISI 1.21 [Hint] 0.9 1.8 3.2 1.6

6 .

The PT is used as a screen for the laboratory evaluation of inherited and acquired deficiencies in the: [Hint] extrinsic pathway of the coagulation cascade intrinsic pathway of the coagulation cascade platelet's function vascular system

7 .

The combination of a prolonged APTT and correction with the mixing study procedure indicates the presence of: [Hint] circulating inhibitor factor VIII deficiency anti-platelet antibodies excessive vitamin K

8 .

Which methodology is used to determine the presence of D-dimers? [Hint] chromogenic assay immunologic assay precipitation assay clot-based assay

Based on the following data, what is the most likely factor deficiency?

. PT APTT TT [Hint] prolonged prolonged normal factor VIII factor V factor IX factor XIII

1 0 .

The bleeding time evaluates all of the following EXCEPT: [Hint] platelet numbers vascular integrity platelet function factor VIII activity Laboratory Testing in Coagulation Level II Multiple Choice

1.

In the APCR assay, why is activated protein C added to the test system? [Hint] to activate factor V activity and shorten the test result to degrade thrombin and prolong the test result to inhibit factor V activity and prolong the test result to enhance factor X activity and shorten the test result

2.

Platelet aggregation studies revealed abnormal aggregation curves with collagen, epinephrine, and ADP, but a normal aggregation curve with ristocetin. Based on these findings, what is the differential diagnosis? [Hint] von Willebrand disease and Bernard-Soulier syndrome Glanzmann's thrombasthenia and von Willebrand disease storage pool disease and Glanzmann's thrombasthenia Bernard-Soulier syndrome and storage pool disease

3.

The following laboratory results are indicative of: Thrombin time: 26.0 seconds Reptilase time: 24.0 seconds [Hint] presence of fibrin degradation products dysfibrinogenemia hypoplasminogenemia presence of heparin

4.

In performing an APCR assay on a patient specimen, you obtained the following results: Standard APTT: 28.4 seconds Modified APTT with APC: 45.3 seconds How would you interpret this assay? [Hint]

presence of factor V Leiden increased levels of activated factor VIII presence of normal factor V decreased levels of antithrombin III

5.

Given the following laboratory results: PT: APTT: normal normal

Platelet aggregation studies: Collagen: abnormal ADP: abnormal Ristocetin: normal Which is the appropriate reflex test? [Hint] ristocetin cofactor assay factor VIII assay bleeding time platelet aggregation study with arachidonic acid

6.

A patient who exhibits delayed bleeding following trauma has the following laboratory data: PT: normal APTT: normal TT: normal: normal What is the most likely clinical condition? [Hint] presence of lupus-like anticoagulant factor XII deficiency increased levels of tissue plasminogen activator factor XIII deficiency

7.

In performing a factor Xa inhibition assay, the clinical laboratory personnel adds less than the appropriate amount of factor Xa to the test system. What effect will this have on the outcome? [Hint] Result will reflect the patient's heparin level, since adequate levels of antithrombin III will be present. Heparin levels will be falsely elevated, since residual factor Xa activity will be decreased. Heparin levels will be falsely decreased, since there were inadequate levels of factor Xa. Result will not reflect the patient's heparin level, since thrombin will be inactivated.

8.

In monitoring a patient on oral anticoagulant therapy, an INR of 2.3 was obtained. How would you interpret this result? [Hint] The patient is adequately anticoagulated and should be tested again in one month. The patient is under-anticoagulated and should be evaluated for a change in dietary habits. The patient is over-anticoagulated and should receive a vitamin K injection. None of the above

9.

Based on the following laboratory data: PT normal

APTT prolonged TT normal Mixing Studies no correction Which is the appropriate reflex test? [Hint] dilute Russell's viper venom time factor XI assay APCR assay FDP assay

10 .

Given the following laboratory data: PT prolonged APTT prolonged TT prolonged Mixing studies correction How would you resolve these results? [Hint] Perform a reptilase time to confirm heparin contamination. Perform a fibrinogen determination to confirm fibrinogen deficiency. Perform a dilute Russell's viper venom time to confirm lupus-like anticoagulant. Perform a hematocrit to confirm a decreased hematocrit (< 20% Automation in Hematology Level I Multiple Choice 1. Which blood cell counting principle is used by the Sysmex SE-9500? [Hint] optical light scatter radio frequency impedance conductivity

2.

In automated blood cell counting instruments, coincidence refers to: [Hint] two or more cells passing through the detection area at the same time leukocytes within the RBC dilution counted as erythrocytes inclusion of nucleated erythrocytes in the leukocyte count electrical interference effecting the impedance count

3.

For the Sysmex SE-9500 instrument, which parameter is derived from a histogram? [Hint] erythrocyte count absolute lymphocyte count leukocyte count MCV

4.

Which dye is used to stain cellular RNA for reticulocyte counting on the Coulter GEN-S instrument? [Hint] acridine orange

new methylene blue oxazine 750 auramine O

5.

For this coagulation instrument, clot formation is detected by a decrease in ball movement as viscosity increases: [Hint] MLA Electra 1800C instrument Organon Teknika Coag-A-Mate XM BBL fibrometer instrument American Bioproducts STArt4 instrument

6.

Both photo-optical and chromogenic principles for coagulation testing are available on the: [Hint] BBL Fibrometer instrument MLA Electra 1800C instrument American Bioproducts STArt 4 instrument Organon Teknika Coag-A-Mate XM instrument

7.

For an instrument using optical light scatter to determine blood cell counts, which cellular characteristic is determined by the degree of side scatter? [Hint] cell size cell granularity nuclear complexity nuclear to cytoplasmic ratio

8.

For the Bayer ADVIA 120, which parameter is directly measured? [Hint] hemoglobin absolute lymphocyte count relative reticulocyte count mean cell volume Which automated blood cell counting instrument uses analysis of the cells volume, conductivity, and light scatter to determine the reticulocyte count? [Hint] Coulter STKS Sysmex SE-9500 + RAM-1 Abbott CELL-DYN 4000 Bayer ADVIA 120

9.

10 .

For the Coulter STKS instrument, which parameter is derived from a histogram or scatterplot? [Hint] absolute monocyte count erythrocyte count platelet count

hematocrit

You might also like